1answer.
Ask question
Login Signup
Ask question
All categories
  • English
  • Mathematics
  • Social Studies
  • Business
  • History
  • Health
  • Geography
  • Biology
  • Physics
  • Chemistry
  • Computers and Technology
  • Arts
  • World Languages
  • Spanish
  • French
  • German
  • Advanced Placement (AP)
  • SAT
  • Medicine
  • Law
  • Engineering
lakkis [162]
3 years ago
5

3.24 x 10-4 8.1 x 10-7

Mathematics
1 answer:
defon3 years ago
7 0
28.4

Explanation

(Multiply the numbers) 3.24 x 10= 32.4
(Subtract the numbers) 32.4 - 4= 28.4
You might be interested in
(8.18x10^-6)(1.15x10^-5)
ladessa [460]

Answer:

9.407x10-11

Step-by-step explanation:

8.18(9.407x(10)−11)10−6(1.15(9.407x(10)−11)10−5)

8.18(9.407x(10)−11)10−6(1.15(9.407x(10)−11)10−5)

8 0
3 years ago
Read 2 more answers
Ava has 34 candy bars. If
aliya0001 [1]

\bf Step-by-step~explanation:

If Ava has 34 candy bars, and each box can hold 5 bars, then we need to find out how many boxes that are filled up.

\bf Step~1:

Divide the number of candy bars (34), by the number each box can hold (5)

\bf\frac{34}{5}=6.8

Since we cannot have 6.8 boxes, we have to round down to 6.

\boxed{\bf Our~final~answer:~6~boxes}

\bf Check~our~answer:

To check our answer, we multiply the number of boxes (6), by the number of bars in each box (5), to get 30. We add Ava's extra bars (4), and we get the number we started off with: 34. This proves our answer is correct!

4 0
3 years ago
What are the 3 steps to problem solving with efficiency?
Burka [1]

Answer:

Step 1: Identify the Problem. ...

Step 2: Analyze the Problem. ...

Step 3: Describe the Problem. ...

Step 4: Look for Root Causes. ...

Step 5: Develop Alternate Solutions. ...

Step 6: Implement the Solution. ...

Step 7: Measure the Results.

7 0
2 years ago
IV
Wewaii [24]

Answer:

1. 648

2. 81

Step-by-step explanation:

1.

6 * 6 * 6 * 3 = 648

2.

3 * 3 * 3 * 3 = 81

5 0
2 years ago
What is b1 and r of geometrical sequence if b3 −b1 = 16 and b5 −b3 = 144.
Naily [24]

Answer:

b1 = 2 ; r = 3

Step-by-step explanation:

Given that :

if b3 −b1 = 16 and b5 −b3 = 144.

For a geometric series :

Ist term = a

Second term = ar

3rd term = ar^2

4th term = ar^3

5th term = ar^4 ;...

If b3 - b1 = 16;

ar^2 - a = 16

a(r^2 - 1) = 16 - - - (1)

b5 - b3 = 144

ar^4 - ar^2 = 144

ar^2(r^2 - 1) = 144 - - - - (2)

Divide (1) by (2)

a(r^2 - 1) / ar^2(r^2 - 1) = 16 /144

a / ar^2 = 1 / 9

ar^2 = 9a

Substitute for a in ar^2 - a = 16

9a - a = 16

8a = 16

a = 2

From ar^2 - a = 16

2r^2 - 2 = 16

2r^2 = 16 + 2

2r^2 = 18

r^2 = 18 / 2

r^2 = 9

r = √9

r = 3

Hence ;

a = b1 = 2 ; r = 3

7 0
3 years ago
Other questions:
  • Solve 4x+2=12 for using the change of base formula logby=logy over logb
    9·2 answers
  • Enter the equation of the line in slope-intercept form.
    14·1 answer
  • If 2+2 is 4 than what is 10+10
    7·2 answers
  • Two choises! Pick the right one!
    7·2 answers
  • What is the inverse of the function f(x)=1/4x-12
    15·1 answer
  • Is this correct? Or wrong
    11·2 answers
  • Which function represents the graph below?
    6·1 answer
  • What would be the integer for this problem <br> ???+-8=-2
    8·2 answers
  • Whats the answer thanks!!!!!
    10·1 answer
  • What is 56x3/4<br> please help
    6·1 answer
Add answer
Login
Not registered? Fast signup
Signup
Login Signup
Ask question!